Blueprint LSAT Prep's ongoing ask-an-instructor extravaganza Forum

Special forum where professionals are encouraged to help law school applicants, students, and graduates.
Post Reply
User avatar
bp shinners

Gold
Posts: 3086
Joined: Wed Mar 16, 2011 7:05 pm

Re: blueprint shinners’ semi-weekly office hours

Post by bp shinners » Wed Jul 31, 2013 2:06 pm

melmoththewanderer wrote: because if as in lines 9-11 these Latin writings are the highest achievements of the Renaissance, it would make logical sense that they would be illuminating comments on their subject matter.
First off, that isn't logically assured. Just because it's the highest achievement doesn't mean it was illuminating. They could be the best of the Renaissance, but still be completely wrong.

Second, the first part is off as well. "Seldom been translated" - I don't know the frequency of the translation, so this is a little too strong. "Inscrutable to modern scholars" is also too strong - they wouldn't be inscrutable to modern Latin scholars because of the lack of translation.

User avatar
neprep

Silver
Posts: 1066
Joined: Fri Jul 26, 2013 11:16 pm

Re: blueprint shinners’ semi-weekly office hours

Post by neprep » Wed Jul 31, 2013 3:20 pm

Hi BP, I had a specific question about PrepTest 36, Section 1, #21. It's a match-the-flaw type question, in which two of the ACs — (B) and (C) — are not even arguments, just conditional statements. Is this common in Parallel questions? Had I known that such kind of wrong ACs exit, I would have confidently eliminated (B) and (C) and not even bothered to analyze them.

Side note: is it always safe to confidently eliminate ACs in parallel-the-reasoning questions if the AC doesn't contain any reasoning in the first place(and the stimulus does)? Just want to make sure.

Thanks! :D

User avatar
bp shinners

Gold
Posts: 3086
Joined: Wed Mar 16, 2011 7:05 pm

Re: blueprint shinners’ semi-weekly office hours

Post by bp shinners » Wed Jul 31, 2013 4:23 pm

neprep wrote:Hi BP, I had a specific question about PrepTest 36, Section 1, #21. It's a match-the-flaw type question, in which two of the ACs — (B) and (C) — are not even arguments, just conditional statements. Is this common in Parallel questions? Had I known that such kind of wrong ACs exit, I would have confidently eliminated (B) and (C) and not even bothered to analyze them.

Side note: is it always safe to confidently eliminate ACs in parallel-the-reasoning questions if the AC doesn't contain any reasoning in the first place(and the stimulus does)? Just want to make sure.

Thanks! :D
First off, I wouldn't say that (B) and (C) lack arguments. The conclusion of (C) is that second clause ("...if the mother's eyes are brown the child's eyes also must be brown."). (B) has more of an implied conclusion, so I guess I agree with you a bit there.

But the important thing is that you compare the conclusions of the ACs to the conclusions of the stimulus. Here, neither (B) nor (C) matches up with the stimulus, so you can throw them out for that reason. (B) can also be thrown out for not really having an explicit conclusion.

To answer your other question, it's not particularly common, but if you have an argument in the stimulus and no argument in the AC, you can definitely rule it out.

User avatar
neprep

Silver
Posts: 1066
Joined: Fri Jul 26, 2013 11:16 pm

Re: blueprint shinners’ semi-weekly office hours

Post by neprep » Wed Jul 31, 2013 4:57 pm

bp shinners wrote:
neprep wrote:Hi BP, I had a specific question about PrepTest 36, Section 1, #21. It's a match-the-flaw type question, in which two of the ACs — (B) and (C) — are not even arguments, just conditional statements. Is this common in Parallel questions? Had I known that such kind of wrong ACs exit, I would have confidently eliminated (B) and (C) and not even bothered to analyze them.

Side note: is it always safe to confidently eliminate ACs in parallel-the-reasoning questions if the AC doesn't contain any reasoning in the first place(and the stimulus does)? Just want to make sure.

Thanks! :D
First off, I wouldn't say that (B) and (C) lack arguments. The conclusion of (C) is that second clause ("...if the mother's eyes are brown the child's eyes also must be brown."). (B) has more of an implied conclusion, so I guess I agree with you a bit there.

But the important thing is that you compare the conclusions of the ACs to the conclusions of the stimulus. Here, neither (B) nor (C) matches up with the stimulus, so you can throw them out for that reason. (B) can also be thrown out for not really having an explicit conclusion.

To answer your other question, it's not particularly common, but if you have an argument in the stimulus and no argument in the AC, you can definitely rule it out.
Great, thanks. I ended up picking (A) ultimately, but I saw an explanation for this question where the only reason given to eliminate (B) and (C) was "Not arguments. Eliminate." And I thought, especially in this case, it couldn't really be that easy. On questions where I was split between a few choices, I find it much more helpful to see why the wrong ACs are wrong.

JuanEighty

New
Posts: 1
Joined: Tue Jul 23, 2013 9:16 pm

Re: blueprint shinners’ semi-weekly office hours

Post by JuanEighty » Thu Aug 01, 2013 8:27 am

When will BP put Preptest 69 on the website?

Want to continue reading?

Register now to search topics and post comments!

Absolutely FREE!


User avatar
bp shinners

Gold
Posts: 3086
Joined: Wed Mar 16, 2011 7:05 pm

Re: blueprint shinners’ semi-weekly office hours

Post by bp shinners » Thu Aug 01, 2013 10:10 am

JuanEighty wrote:When will BP put Preptest 69 on the website?
They go up when it's cleared for licensing and we feel it would be most beneficial for our students to take the exam. I don't have an exact date, but I would expect it in late August/September.

jmjm

Bronze
Posts: 329
Joined: Mon Mar 28, 2011 1:59 am

Re: blueprint shinners’ semi-weekly office hours

Post by jmjm » Fri Aug 09, 2013 3:18 pm

Interested in thoughts about original post at http://www.top-law-schools.com/forums/v ... 6&t=214576

melmoththewanderer

New
Posts: 86
Joined: Sun Apr 14, 2013 12:31 pm

Re: blueprint shinners’ semi-weekly office hours

Post by melmoththewanderer » Fri Aug 09, 2013 7:52 pm

BP! My plan is to catch you off guard today not with the usual RC question, but with two LR questions.

So I wanted to ask about PT39-S2-Q6 and PT39-S2-Q16.

For 6, why is A incorrect? For letter D, I looked up mundane and it means uninteresting, which doesn't include complexity in its scope. Also, the stimulus is limited only to mundane ideas... we have no idea what this author thinks of interesting ideas, so how can we defend D?

For 16, I whittled it down to C and D. I thought the definition of assumption is that it is unstated. But here, it is right there, in black and white. So how could that be an assumption? Also why isn't it C?

Thanks!

jmjm

Bronze
Posts: 329
Joined: Mon Mar 28, 2011 1:59 am

Re: blueprint shinners’ semi-weekly office hours

Post by jmjm » Sun Aug 11, 2013 4:06 am

hey bp!
pt-39 lr-2 q22 seems confusing. E contains fallacious attack on the drama critics that says nothing about the truth of the argument. B should not weaken since past occurrence is not indicative of future occurrence. Any reason why both weaken the argument?

Want to continue reading?

Register for access!

Did I mention it was FREE ?


User avatar
bp shinners

Gold
Posts: 3086
Joined: Wed Mar 16, 2011 7:05 pm

Re: blueprint shinners’ semi-weekly office hours

Post by bp shinners » Mon Aug 12, 2013 1:53 pm

jmjm wrote:Interested in thoughts about original post at http://www.top-law-schools.com/forums/v ... 6&t=214576
Sufficient assumption - if you add it to the premises of the stimulus, the conclusion is 100% certain to follow, i.e. you have a valid argument.

Necessary assumption - if you negate it/say it isn't true, then the premises have no chance of supporting the conclusion, i.e. your argument is invalid unless you start fresh with a new set of premises.

User avatar
bp shinners

Gold
Posts: 3086
Joined: Wed Mar 16, 2011 7:05 pm

Re: blueprint shinners’ semi-weekly office hours

Post by bp shinners » Mon Aug 12, 2013 2:07 pm

melmoththewanderer wrote:So I wanted to ask about PT39-S2-Q6 and PT39-S2-Q16.

For 6, why is A incorrect? For letter D, I looked up mundane and it means uninteresting, which doesn't include complexity in its scope. Also, the stimulus is limited only to mundane ideas... we have no idea what this author thinks of interesting ideas, so how can we defend D?
For (A), the author never makes an absolute statement about complexity and better writing. He makes a relative statement about overly pretentious writing to convey simple/mundane ideas. So we can't pick (A) because we don't know that simpler writing is always better - just when it matches the mundane ideas being expressed.

"Mundane" includes a level of simplicity in the definition. Also, people read (D) to be a pretty strong statement, but it's not, really. It's saying that writing shouldn't be more complex than the ideas expressed. It doesn't say "as complex" or "not complex". Instead, it's creating a relative limit. So when the author here concludes with a statement about writing not being more complex than the ideas expressed, he's just saying that even if we have interesting ideas, the writing shouldn't be more complex than those ideas. It can be as complex, or less complex.
For 16, I whittled it down to C and D. I thought the definition of assumption is that it is unstated. But here, it is right there, in black and white. So how could that be an assumption? Also why isn't it C?

Thanks!
The definition of an assumption is something that is taken for granted as true. When I have just a single person speaking, then an assumption has to be unstated. However, once I get multiple viewpoints, the assumption of one argument can be stated by the speaker of the other argument. Here, we have the author and the biologists. The author points out the baseline belief of biologists, but it only states that "biologists agree" that frogs are related to these fish. That's not the same as saying that the frogs are related to these fish (we'd need the author to say something like, "Biologists have proven that..."). The biologists are taking it as true for their argument, but we have no proof that it's true, so we can classify it as an assumption.

(C) is out because no one makes the contention that human beings must be descended from either lungfish or coelacanths. Stevens-Hoyt makes the claim about lungfish, and Grover makes the claim about coelacanths, but no one ever comes in to say that it must be one of these two. For all we know, there could be scientists out there who think that it's a completely different fish.

User avatar
bp shinners

Gold
Posts: 3086
Joined: Wed Mar 16, 2011 7:05 pm

Re: blueprint shinners’ semi-weekly office hours

Post by bp shinners » Mon Aug 12, 2013 2:19 pm

jmjm wrote:hey bp!
pt-39 lr-2 q22 seems confusing. E contains fallacious attack on the drama critics that says nothing about the truth of the argument. B should not weaken since past occurrence is not indicative of future occurrence. Any reason why both weaken the argument?
(B) - If I'm talking just odds of something happening, then yes, a temporal fallacy comes into play. And if I was trying to kill the argument, then (B) wouldn't work. However, I'm just trying to weaken the argument. And if a company staging the play has always put on dull performances, that makes it less likely that they will suddenly develop acting talent and put on a good play. The proof here isn't that they've never done it, so they won't do it. The proof here is that they most likely don't have good actors, so they won't do it.

(E) - This isn't an ad hominem attack - it's telling me about the interaction between critics and the audience. The critic concludes that audiences will enjoy Warner's latest play. Well, if audiences' views don't normally align with that of critics, it makes it less likely that the critic's view of audiences will be correct. It'd be like if I said, "My girlfriend is going to love this TV show!" and an answer choice said, "bp shinners is usually wrong about his girlfriend's taste in TV shows." It would definitely make you question whether or not my conclusion is correct.

jmjm

Bronze
Posts: 329
Joined: Mon Mar 28, 2011 1:59 am

Re: blueprint shinners’ semi-weekly office hours

Post by jmjm » Mon Aug 12, 2013 3:53 pm

bp shinners wrote:
jmjm wrote:Interested in thoughts about original post at http://www.top-law-schools.com/forums/v ... 6&t=214576
Sufficient assumption - if you add it to the premises of the stimulus, the conclusion is 100% certain to follow, i.e. you have a valid argument.

Necessary assumption - if you negate it/say it isn't true, then the premises have no chance of supporting the conclusion, i.e. your argument is invalid unless you start fresh with a new set of premises.
A more specific example of what I am referring to in the post is pt-35, s1, q14 in which necessary assumption is the missing link. Quite likely this issue is not the difference between necessary and sufficient assumption based on ps lrb and manhattan guide. Also discussed on page 273 PS LRB under 'assumptions and conditionality.'

Register now!

Resources to assist law school applicants, students & graduates.

It's still FREE!


jmjm

Bronze
Posts: 329
Joined: Mon Mar 28, 2011 1:59 am

Re: blueprint shinners’ semi-weekly office hours

Post by jmjm » Mon Aug 12, 2013 4:13 pm

bp shinners wrote:
jmjm wrote:hey bp!
pt-39 lr-2 q22 seems confusing. E contains fallacious attack on the drama critics that says nothing about the truth of the argument. B should not weaken since past occurrence is not indicative of future occurrence. Any reason why both weaken the argument?
(B) - If I'm talking just odds of something happening, then yes, a temporal fallacy comes into play. And if I was trying to kill the argument, then (B) wouldn't work. However, I'm just trying to weaken the argument. And if a company staging the play has always put on dull performances, that makes it less likely that they will suddenly develop acting talent and put on a good play. The proof here isn't that they've never done it, so they won't do it. The proof here is that they most likely don't have good actors, so they won't do it.

(E) - This isn't an ad hominem attack - it's telling me about the interaction between critics and the audience. The critic concludes that audiences will enjoy Warner's latest play. Well, if audiences' views don't normally align with that of critics, it makes it less likely that the critic's view of audiences will be correct. It'd be like if I said, "My girlfriend is going to love this TV show!" and an answer choice said, "bp shinners is usually wrong about his girlfriend's taste in TV shows." It would definitely make you question whether or not my conclusion is correct.
Good to know the fine point about how in B the temporally fallacious claim can still weaken the argument but not kill it due to it being fallacious.
E still makes my engineering mind go thinking wouldn't the analogy be closer to E if the answer came "the girlfriend doesn't find bp's taste in tv show good". In other words, wouldn't it be a fallacy in logic to go from "A thinks B is unreliable" to "B's opinion or argument about A is wrong".
So it seems just because E is fallacious doesn't mean it doesn't weaken the argument, right.

User avatar
bp shinners

Gold
Posts: 3086
Joined: Wed Mar 16, 2011 7:05 pm

Re: blueprint shinners’ semi-weekly office hours

Post by bp shinners » Tue Aug 13, 2013 2:44 pm

jmjm wrote:
bp shinners wrote:
jmjm wrote:Interested in thoughts about original post at http://www.top-law-schools.com/forums/v ... 6&t=214576
Sufficient assumption - if you add it to the premises of the stimulus, the conclusion is 100% certain to follow, i.e. you have a valid argument.

Necessary assumption - if you negate it/say it isn't true, then the premises have no chance of supporting the conclusion, i.e. your argument is invalid unless you start fresh with a new set of premises.
A more specific example of what I am referring to in the post is pt-35, s1, q14 in which necessary assumption is the missing link. Quite likely this issue is not the difference between necessary and sufficient assumption based on ps lrb and manhattan guide. Also discussed on page 273 PS LRB under 'assumptions and conditionality.'
I'm not sure what you're getting at, but I never said that a necessary assumption can't also be sufficient - just that it doesn't have to be. If the necessary assumption is the missing link, however, it will be because the argument structure is set up in a way that forming that particular missing link, as you call it, is the only way to get to the conclusion based on the premises as presented.

User avatar
bp shinners

Gold
Posts: 3086
Joined: Wed Mar 16, 2011 7:05 pm

Re: blueprint shinners’ semi-weekly office hours

Post by bp shinners » Tue Aug 13, 2013 2:46 pm

jmjm wrote: In other words, wouldn't it be a fallacy in logic to go from "A thinks B is unreliable" to "B's opinion or argument about A is wrong".
So it seems just because E is fallacious doesn't mean it doesn't weaken the argument, right.
I didn't re-read the question, so I might be wrong, but I don't think the argument relies on "B's opinion/argument about A" being wrong - just proof that it might be wrong. If the audiences find the critics unreliable, then it makes me think that the critic's view of the audience isn't correct. What the critic said still might be right, but it's more likely that it's wrong now that I know they commonly disagree.

User avatar
bp shinners

Gold
Posts: 3086
Joined: Wed Mar 16, 2011 7:05 pm

Re: blueprint shinners’ semi-weekly office hours

Post by bp shinners » Wed Aug 21, 2013 4:05 pm

Open for business, TLS. But I just had rotator cuff surgery, so I'm a little loopy from the pain meds, and I am "typing" with Dragon NaturallySpeaking, so there might be some typos.

Get unlimited access to all forums and topics

Register now!

I'm pretty sure I told you it's FREE...


User avatar
neprep

Silver
Posts: 1066
Joined: Fri Jul 26, 2013 11:16 pm

Re: blueprint shinners’ semi-weekly office hours

Post by neprep » Wed Aug 21, 2013 4:30 pm

I know it's been said before, but do you agree with the proposition that the LR sections from older PTs and those from the newer ones might be suited to different strengths? I was consistently around -2 on newer LR sections, but SuperPrep A and B suddenly had me at -4 and -5. Now, -3 and -4 doesn't sound like an egregious variation, but -2 and -6 does. I wonder if the language used in modern-day LR is simply clearer and the task overall more predictable.

While we're at it, anywho, do you have any advice on how to deal with formal logic questions with quantifiers quickly? Case in point: SuperPrep A, Section 4, Question 24. I see why the right answer is correct — and anybody will, given enough time — but under timed pressure this question is like a black hole.

Thanks!

User avatar
bp shinners

Gold
Posts: 3086
Joined: Wed Mar 16, 2011 7:05 pm

Re: blueprint shinners’ semi-weekly office hours

Post by bp shinners » Wed Aug 21, 2013 4:51 pm

neprep wrote:I know it's been said before, but do you agree with the proposition that the LR sections from older PTs and those from the newer ones might be suited to different strengths? I was consistently around -2 on newer LR sections, but SuperPrep A and B suddenly had me at -4 and -5. Now, -3 and -4 doesn't sound like an egregious variation, but -2 and -6 does. I wonder if the language used in modern-day LR is simply clearer and the task overall more predictable.

While we're at it, anywho, do you have any advice on how to deal with formal logic questions with quantifiers quickly? Case in point: SuperPrep A, Section 4, Question 24. I see why the right answer is correct — and anybody will, given enough time — but under timed pressure this question is like a black hole.

Thanks!
I think the writers of the LSAT have gotten much better at being very precise with their language since they first started. That's what happens when you can test your language on tens of thousands of people every year - you start to realize the ways certain people will interpret certain words, and in ways that you never even thought of.

As far as quantifiers, they tend to show up on the LSAT in one of two ways. The first is as a quick method of elimination in parallel and certain parallel flaw questions. If you see quantifiers in a parallel or parallel flaw question, absolutely rely on them to quickly get through the answer choices. If this happens, you should jump for joy. The second method is in implication questions that ask you to combine quantified statements. This is likely to come up only once or twice on any given exam. The best way around this is to just sit down and memorize the quantified statements that can, in fact, be combined. If you try to derive it each time it comes up, it's going to take forever entered a mess up at some point.

As far as the valid combinations, I go with the following rule of thumb. First, before the rule of thumb, you can never combine a most and a some statement, or a some and a some statement. Any other combination you can do if the sufficient condition of the stronger statement is shared. I call this the three Ss - sufficient, stronger, shared. Using this, you almost always get another some statement; the one exception is when you have an all and a most statement where the sufficient condition of the all statement is in the necessary condition of the most statement. If that's the case, you get a new most statement.

User avatar
neprep

Silver
Posts: 1066
Joined: Fri Jul 26, 2013 11:16 pm

Re: blueprint shinners’ semi-weekly office hours

Post by neprep » Wed Aug 21, 2013 5:05 pm

bp shinners wrote:
neprep wrote:I know it's been said before, but do you agree with the proposition that the LR sections from older PTs and those from the newer ones might be suited to different strengths? I was consistently around -2 on newer LR sections, but SuperPrep A and B suddenly had me at -4 and -5. Now, -3 and -4 doesn't sound like an egregious variation, but -2 and -6 does. I wonder if the language used in modern-day LR is simply clearer and the task overall more predictable.

While we're at it, anywho, do you have any advice on how to deal with formal logic questions with quantifiers quickly? Case in point: SuperPrep A, Section 4, Question 24. I see why the right answer is correct — and anybody will, given enough time — but under timed pressure this question is like a black hole.

Thanks!
I think the writers of the LSAT have gotten much better at being very precise with their language since they first started. That's what happens when you can test your language on tens of thousands of people every year - you start to realize the ways certain people will interpret certain words, and in ways that you never even thought of.

As far as quantifiers, they tend to show up on the LSAT in one of two ways. The first is as a quick method of elimination in parallel and certain parallel flaw questions. If you see quantifiers in a parallel or parallel flaw question, absolutely rely on them to quickly get through the answer choices. If this happens, you should jump for joy. The second method is in implication questions that ask you to combine quantified statements. This is likely to come up only once or twice on any given exam. The best way around this is to just sit down and memorize the quantified statements that can, in fact, be combined. If you try to derive it each time it comes up, it's going to take forever entered a mess up at some point.

As far as the valid combinations, I go with the following rule of thumb. First, before the rule of thumb, you can never combine a most and a some statement, or a some and a some statement. Any other combination you can do if the sufficient condition of the stronger statement is shared. I call this the three Ss - sufficient, stronger, shared. Using this, you almost always get another some statement; the one exception is when you have an all and a most statement where the sufficient condition of the all statement is in the necessary condition of the most statement. If that's the case, you get a new most statement.
So suppose:

1. Most, but not all, Blueprint Instructors are happy people
2. Most, but not all, Blueprint Instructors are aliens

Are both of the following valid inference from (1) and (2)?

-Some happy people are aliens
-Some happy people are not aliens (I think this is invalid. The same "most" of the pool of BP instructors can both be happy and be alines, leaving the minority of the pool being neither happy nor extraterrestrial; right?)

User avatar
bp shinners

Gold
Posts: 3086
Joined: Wed Mar 16, 2011 7:05 pm

Re: blueprint shinners’ semi-weekly office hours

Post by bp shinners » Wed Aug 21, 2013 5:29 pm

neprep wrote: So suppose:

1. Most, but not all, Blueprint Instructors are happy people
2. Most, but not all, Blueprint Instructors are aliens
This is actually slightly interesting (to an LSAT nerd, at least) because each of these creates to quantified statements:
Most Blueprint instructors are happy people; some Blueprint instructors are not happy people.
Most Blueprint instructors are aliens; some Blueprint instructors are not aliens.

I can definitely combine the two most statements to get, as you stated, some happy people are aliens. Any other combination, however, would involve the combination of a most and a some statement, or to some statements. Neither of those is allowed, so we can't come up with any other valid statements.
-Some happy people are not aliens (I think this is invalid. The same "most" of the pool of BP instructors can both be happy and be alines, leaving the minority of the pool being neither happy nor extraterrestrial; right?)
Correct.

Communicate now with those who not only know what a legal education is, but can offer you worthy advice and commentary as you complete the three most educational, yet challenging years of your law related post graduate life.

Register now, it's still FREE!


User avatar
crestor

Bronze
Posts: 313
Joined: Thu Jan 10, 2013 5:37 am

Re: blueprint shinners’ semi-weekly office hours

Post by crestor » Wed Aug 21, 2013 6:44 pm

Hey BP,

Once again your advise is gold. Took 3 days off (Friday, Saturday, and Sunday) and hit on of my highest PTS after slight review on Monday, Tuesday. I feel like I should be paying you. Congratz on getting engaged btw!

User avatar
bp shinners

Gold
Posts: 3086
Joined: Wed Mar 16, 2011 7:05 pm

Re: blueprint shinners’ semi-weekly office hours

Post by bp shinners » Wed Aug 21, 2013 7:06 pm

crestor wrote:Hey BP,

Once again your advise is gold. Took 3 days off (Friday, Saturday, and Sunday) and hit on of my highest PTS after slight review on Monday, Tuesday. I feel like I should be paying you. Congratz on getting engaged btw!
Thanks! And you can always send Laphroig ;-)

Seriously, though, congrats on the rebound! Keep it up.

magickware

Bronze
Posts: 359
Joined: Sun Nov 04, 2012 1:27 pm

Re: blueprint shinners’ semi-weekly office hours

Post by magickware » Wed Aug 21, 2013 7:30 pm

BP, I have a question regarding PT 53, Sec.4, Q.21.

I narrowed it down to C and E, but I can't quite figure out conclusively why C is a better answer choice.

Is it because we're not told that it would be safer to move the hazardous waste, and as such we can only talk about how moving it would reduce the threat in comparison to keeping it there?

But what about the words "contain it safely" in the stimulus? While I understand that means that both the current location and the new location are not safe, wouldn't the entire argument create the implication that the new location is safer than the old one?

Or am I over-thinking this greatly and E is wrong because of the word "any"?

User avatar
bp shinners

Gold
Posts: 3086
Joined: Wed Mar 16, 2011 7:05 pm

Re: blueprint shinners’ semi-weekly office hours

Post by bp shinners » Wed Aug 21, 2013 8:03 pm

magickware wrote: While I understand that means that both the current location and the new location are not safe, wouldn't the entire argument create the implication that the new location is safer than the old one?
That's exactly what answer choice (C) says!

I know that the current site clearly poses unacceptable risks; I know the objections are based on extremely implausible scenarios. I wouldn't pick (C) for a must be true question, but the implication is clearly there for a most strongly supported question that the new site is, at least a little bit, safer than the current site.
Or am I over-thinking this greatly and E is wrong because of the word "any"?
That "any" is exactly why answer choice (E) is wrong. I don't say that the current location is the least safe site, which is what I need before I could pick answer choice (E). Really strong answer choices are rarely correct in most strongly supported questions.

Seriously? What are you waiting for?

Now there's a charge.
Just kidding ... it's still FREE!


Post Reply

Return to “Free Help and Advice from Professionals”